orthocomplemented lattices with a symmetric difference
play

Orthocomplemented lattices with a symmetric difference Milan Matou - PowerPoint PPT Presentation

Orthocomplemented lattices with a symmetric difference Milan Matou sek 1 ODLs 2 Z 2 -valued states 3 The variety OML 1 ODLs Definition 1.1 Let L = ( X, , , , 0 , 1 , ), where ( X, , , , 0 , 1) is an


  1. Orthocomplemented lattices with a symmetric difference Milan Matouˇ sek 1 ODLs 2 Z 2 -valued states 3 The variety OML △

  2. 1 ODLs Definition 1.1 Let L = ( X, ∧ , ∨ , ⊥ , 0 , 1 , △ ), where ( X, ∧ , ∨ , ⊥ , 0 , 1) is an orthocomplemented lattice and △ : X 2 → X is a binary operation. Then L is said to be an orthocomplemented difference lattice (abbr., an ODL) if the following formulas hold in L : (D 1 ) x △ ( y △ z ) = ( x △ y ) △ z , (D 2 ) x △ 1 = x ⊥ , 1 △ x = x ⊥ , (D 3 ) x △ y ≤ x ∨ y . ( X, ∧ , ∨ , ⊥ , 0 , 1) will be denoted by L supp and called the support of L .

  3. Proposition 1.2 Let L = ( X, ∧ , ∨ , ⊥ , 0 , 1 , △ ) be an ODL. Then the following statements hold true: (1) x △ 0 = x , 0 △ x = x , (2) x △ x = 0 , (3) x △ y = y △ x , (4) x △ y ⊥ = x ⊥ △ y = ( x △ y ) ⊥ , (5) x ⊥ △ y ⊥ = x △ y , (6) x △ y = 0 ⇔ x = y , (7) ( x ∧ y ⊥ ) ∨ ( y ∧ x ⊥ ) ≤ x △ y ≤ ( x ∨ y ) ∧ ( x ∧ y ) ⊥ .

  4. Theorem 1.3 Let L be an ODL. Then its support L supp is an OML. Proof. We will prove x, y ∈ L, x ≤ y, y ∧ x ⊥ = 0 ⇒ x = y . x ≤ y ⇒ ( x ∧ y ⊥ ) ∨ ( y ∧ x ⊥ ) = y ∧ x ⊥ = 0, ( x ∨ y ) ∧ ( x ∧ y ) ⊥ = y ∧ x ⊥ = 0. Prop. 1.2: x △ y = 0 (by (7)) and therefore x = y (by (6)). Let L be an ODL. Then any OML notion can be referred to L as well by applying this notion to the corresponding OML L supp .

  5. Proposition 1.4 Let L be an ODL, a, b ∈ L . (i) a C b ⇒ a △ b = ( a ∧ b ⊥ ) ∨ ( b ∧ a ⊥ ) = ( a ∨ b ) ∧ ( a ∧ b ) ⊥ ; (A corollary: For each block B of L , the operation △ on L acts on B as the standard symmetric difference.) (ii) a ⊥ b (i.e. a ≤ b ⊥ ) ⇔ a △ b = a ∨ b .

  6. A NATURAL QUESTION: Which OMLs are embed- dable into ODLs ? A MORE GENERAL QUESTION: Let us consider the variety OML △ of OMLs generated by the class { L supp ; L is an ODL } , i.e. OML △ = HSP ( { L supp ; L is an ODL } ). Which OMLs belong to the variety OML △ ? The main aim of this note is to show that the class OML △ does not contain all OMLs.

  7. Proof of Prop. 1.2. (D 2 ): 1 △ 1 = 1 ⊥ = 0. (1) x △ 0 = x △ (1 △ 1) = ( x △ 1) △ 1 = x ⊥ △ 1 = ( x ⊥ ) ⊥ = x ; the identity 0 △ x = x has an analogous proof. (2) Let us first show that x ⊥ △ x ⊥ = x △ x . We con- secutively obtain x ⊥ △ x ⊥ = ( x △ 1) △ (1 △ x ) = ( x △ (1 △ 1)) △ x = ( x △ 0) △ x = x △ x . Moreover, we have x △ x ≤ x as well as x △ x = x ⊥ △ x ⊥ ≤ x ⊥ . This implies that x △ x ≤ x ∧ x ⊥ = 0. (3) ( x △ y ) △ ( x △ y ) = 0 ⇒ ( x △ y △ x △ y ) △ y = 0 △ y ⇒ x △ y △ x = y ⇒ ( x △ y △ x ) △ x = y △ x ⇒ x △ y = y △ x . (4) x △ y ⊥ = x △ ( y △ 1) = ( x △ y ) △ 1 = ( x △ y ) ⊥ . (5) Applying (4), we obtain x ⊥ △ y ⊥ = ( x ⊥ △ y ) ⊥ = ( x △ y ) ⊥⊥ = x △ y . (6) If x = y , then x △ y = 0 by the condition (2). Con- versely, suppose that x △ y = 0. Then x = x △ 0 =

  8. x △ ( y △ y ) = ( x △ y ) △ y = 0 △ y = y . (7) The property (D 3 ) together with the properties (4), (5) imply that x △ y ≤ x ∨ y , x △ y ≤ x ⊥ ∨ y ⊥ = ( x ∧ y ) ⊥ , x ∧ y ⊥ ≤ x △ y , x ⊥ ∧ y ≤ x △ y . Proof of Prop. 1.4. (i) It follows from Prop. 1.2. (ii) ( ⇒ ) We use Prop. 1.2. ( ⇐ ) x △ y = x ∨ y ⇒ y ≤ x △ y ⊥ ≤ y ⊥ . But also y ⊥ ≤ y ⊥ . x △ y ⇒ (D 3 ): ( x △ y ⊥ ) △ y ⊥ ≤ y ⊥ . Finally, ( x △ y ⊥ ) △ y ⊥ = x △ ( y ⊥ △ y ⊥ ) = x △ 0 = x .

  9. 2 Z 2 -valued states Z 2 ≡ the group { 0 , 1 } with the modulo 2 addition ⊕ Definition 2.1 Let L be an OML. (i) Let s : L → Z 2 be a mapping. Then s is said to be a Z 2 - valued state (abbr., a Z 2 - state ) provided s (1 L ) = 1 and s ( x ∨ y ) = s ( x ) ⊕ s ( y ) whenever x, y ∈ L , x ≤ y ⊥ . (ii) L is called Z 2 - full if ∀ x, y ∈ L, x � = y, x � = 0 L , y � = 1 L ∃ Z 2 -state s on L : s ( x ) = 1 & s ( y ) = 0.

  10. Definition 2.2 Let L be an ODL and let I ⊆ L . I is a △ - ideal if 0 L ∈ I & ∀ a, b ∈ I : a △ b ∈ I ; I is a proper △ - ideal if I is a △ -ideal and 1 L �∈ I ; I is a maximal △ - ideal if I is proper △ -ideal and for any proper △ -ideal J with I ⊆ J we have I = J .

  11. Lemma 2.3 Suppose that L is an ODL and I is a proper △ -ideal in L . Suppose that x ∈ L and neither x nor x ⊥ belongs to I . Let us write J = I ∪ { a △ x ; a ∈ I } . Then J is also a proper △ -ideal in L and, moreover, x ∈ J and x ⊥ �∈ J . Proposition 2.4 Let L be an ODL and let I be a proper △ -ideal in L . Then I is a maximal △ -ideal in L iff card( { x, x ⊥ } ∩ I ) = 1 for any x ∈ L . Proposition 2.5 (Stone’s lemma) Let L be an ODL, let I 0 be a proper △ -ideal in L and let b ∈ L , b �∈ I 0 . Then there is a maximal △ -ideal I such that I 0 ⊆ I and b �∈ I .

  12. Proposition 2.6 Let L be an ODL and I be a maximal △ -ideal in L . Let us define a mapping s : L → Z 2 as follows: s ( a ) = 0 (resp., s ( a ) = 1 ) if a ∈ I (resp., a �∈ I ). Then s ( x △ y ) = s ( x ) ⊕ s ( y ) for any x, y ∈ L . A consequence: The mapping s is a Z 2 -state on L supp . Theorem 2.7 Let L be an ODL. Then its support L supp is Z 2 -full.

  13. Proof of Lemma 2.3. The set J is obviously a △ -ideal. Let us see that 1 L �∈ J . Suppose on the contrary that 1 L ∈ J . Then 1 L = a △ x for some element a ∈ I . The But x ⊥ does equality 1 L = a △ x implies that a = x ⊥ . not belong to I which is a contradiction. Thus, 1 L �∈ J . Further, x = 0 L △ x ∈ J . If x ⊥ ∈ J , then 1 L = x △ x ⊥ ∈ J – a contradiction again. ( ⇒ ) Suppose that I is maximal Proof of Prop. 2.4. and x ∈ L . Suppose further that x �∈ I and, also, x ⊥ �∈ I . Then (Lemma 2.3) there is a △ -ideal J such that I ⊆ J and I � = J . As a result, at least one element of the set { x, x ⊥ } belongs to I . Looking for a contradiction, sup- Then x △ x ⊥ = 1 which means pose that { x, x ⊥ } ⊆ I . that 1 ∈ I – a contradiction ( I is supposed proper). ( ⇐ ) Let us suppose that card( { x, x ⊥ } ∩ I ) = 1 for any

  14. x ∈ L . Suppose further that I ⊂ J for a proper △ -ideal J . J is strictly larger than I ⇒ ∃ b ∈ L : b ∈ J, b �∈ I . card( { b, b ⊥ } ∩ I ) = 1 and b �∈ I ⇒ b ⊥ ∈ I . Now, both the elements b and b ⊥ belong to J and there- fore 1 L = b △ b ⊥ ∈ J . This means that J is not proper. Proof of Prop. 2.5. Write I = { J ⊆ L ; J is a proper △ -ideal, I 0 ⊆ J and b �∈ J } . Then I 0 ∈ I and therefore I � = ∅ . Zorn’s lemma: the set I ordered by inclusion contains a maximal element, I . (1) I is a proper △ -ideal, I 0 ⊆ I and b �∈ I . (2) b ⊥ ∈ I (otherwise the △ -ideal I ′ = I ∪ { c △ b ⊥ ; c ∈ I } extends I , Lemma 2.3, and I ′ belongs to the system I ). (3) I is a maximal △ -ideal (Suppose therefore that I ⊂ J

  15. for a proper △ -ideal J . Thus, J is strictly larger than I and therefore J �∈ I . Therefore b ∈ J and since b ⊥ ∈ I ⊆ J , we see that 1 L = b △ b ⊥ ∈ J . This means that J is not proper.)

  16. Proof of Prop. 2.6. Let us consider x, y ∈ L . We are to prove the equality s ( x △ y ) = s ( x ) ⊕ s ( y ). We will argue by cases. For example, let us suppose that x ∈ I and y �∈ I . Then x △ y �∈ I (indeed, should x △ y be an element of I , then y = x △ ( x △ y ) ∈ I which is a contradiction). Hence s ( x △ y ) = 1 = 0 ⊕ 1 = s ( x ) ⊕ s ( y ). It remains to show that s is a Z 2 -state on L supp . Of course, s (1) = 1. Let us take x, y ∈ L with x ⊥ y . Prop. 1.4: x △ y = x ∨ y . Then s ( x ∨ y ) = s ( x △ y ) = s ( x ) ⊕ s ( y ) by the analysis above. The proof of Prop. 2.6 is complete.

  17. Let L be an ODL. Let x, y ∈ L Proof of Thm. 2.7. with x � = y , x � = 0 and y � = 1. Let us set I 0 = { 0 L , y } . According to Prop. 2.4 there is a maximal △ -ideal I in L such that y ∈ I and x �∈ I . Let us set s ( a ) = 0 for a ∈ I and s ( a ) = 1 for a ∈ L , a �∈ I . Then, according to Prop. 2.6, the mapping s is a Z 2 -state on L supp .

  18. 3 The variety OML △ Let us recall that OML △ = HSP ( { L supp ; L ∈ ODL} ). Theorem 3.1 Let K ∈ OML △ and let x 0 ∈ K , x 0 � = 1 K . Then there is a Z 2 -state, s , on K such that s ( x 0 ) = 0 .

  19. Proposition 3.2 Suppose that L is an OML. Suppose that there are blocks B 1 , B 2 , . . . , B n of L such that the following two conditions are satisfied: (1) each B i , 1 ≤ i ≤ n is finite and n is an odd number, (2) if a ∈ L is an atom in L , then a lies in an even number of blocks B 1 , B 2 , . . . , B n (i.e., the cardinality of the set { i ; a ∈ B i } is even). Then there is no Z 2 -state on L . Proof. Let s : L → Z 2 be a Z 2 -state. Then ⊕ a ∈ At( B i ) s ( a ) = s (1 L ) = 1 ( i = 1 , . . . , n ). Therefore ⊕ i ∈{ 1 ,...,n } ( ⊕ a ∈ At( B i ) s ( a )) = 1 ⊕ . . . ⊕ 1. The right-hand side = 1 (contains the element 1 exactly n -many times and n is odd). The left-hand side = 0 (contains any expression s ( a ) (where a ∈ At( A )) even number of times).

Download Presentation
Download Policy: The content available on the website is offered to you 'AS IS' for your personal information and use only. It cannot be commercialized, licensed, or distributed on other websites without prior consent from the author. To download a presentation, simply click this link. If you encounter any difficulties during the download process, it's possible that the publisher has removed the file from their server.

Recommend


More recommend